Find Value of arccot(pi/4): Explanation & Solution

Click For Summary
The discussion revolves around finding the value of arccot(pi/4). The original poster is confused about the relationship between inverse cotangent and inverse tangent, mistakenly interpreting the unit circle value as 1. Responses clarify that arccot(pi/4) corresponds to arctan(0.7854), and emphasize that the angle should be in the first quadrant, not the fourth. The correct value of arccot(pi/4) is approximately 0.7854, aligning with the tangent of pi/4. Understanding the distinction between cotangent and tangent is crucial for solving this problem accurately.
Fellowroot
Messages
92
Reaction score
0

Homework Statement



Fin the value of arccot(pi/4)

Homework Equations



unit circle

The Attempt at a Solution



I honestly can't believe that I'm stuck on this as this shouldn't stump me.

My logic is that since its inverse cotangent then its related to inverse tangent and so the value that this reduces to will be found in quadrant 1 or 4 only.

The value I get reading the unit circle is 1, but my calculator says 1.502 and wolframalpha says 0.905.

Can anyone help explain this? Thanks.
 
Physics news on Phys.org
Fellowroot said:

Homework Statement



Fin the value of arccot(pi/4)

Homework Equations



unit circle

The Attempt at a Solution



I honestly can't believe that I'm stuck on this as this shouldn't stump me.

My logic is that since its inverse cotangent then its related to inverse tangent and so the value that this reduces to will be found in quadrant 1 or 4 only.

The value I get reading the unit circle is 1, but my calculator says 1.502 and wolframalpha says 0.905.

Can anyone help explain this? Thanks.

If you get 1 it means that you took the tangent of pi/4.

You need the arc cotangent of pi/4, that is arctan(0.7854).

If the tangent/cotangent is positive the angle is either in the first quadrant, or ? (not in the fourth).

ehild
 
Question: A clock's minute hand has length 4 and its hour hand has length 3. What is the distance between the tips at the moment when it is increasing most rapidly?(Putnam Exam Question) Answer: Making assumption that both the hands moves at constant angular velocities, the answer is ## \sqrt{7} .## But don't you think this assumption is somewhat doubtful and wrong?

Similar threads

Replies
3
Views
2K
Replies
15
Views
3K
  • · Replies 15 ·
Replies
15
Views
2K
  • · Replies 5 ·
Replies
5
Views
2K
Replies
9
Views
2K
  • · Replies 6 ·
Replies
6
Views
2K
Replies
3
Views
3K
  • · Replies 11 ·
Replies
11
Views
3K
  • · Replies 4 ·
Replies
4
Views
13K
  • · Replies 3 ·
Replies
3
Views
1K